Download as pdf or txt
Download as pdf or txt
You are on page 1of 57

FAC2601

LEARNING UNIT 7

LEASES – IFRS 16

Financial Accounting
for Companies
CONTENTS
Page
LEARNING UNIT 7: LEASES – IFRS 16 .............................................................................. 1
7.1 Schematic representation of IFRS 16 ................................................................... 4
7.2 Reporting Framework (IFRS & IFRS for Small and medium size enterprises) ...... 5
7.3 Identifying a lease ................................................................................................. 6
7.3.1 Steps to follow to determine whether a contract contains a lease ..................... 7
7.3.2 Separating components of a contract .................................................................. 10
7.3.3 Lease term ............................................................................................................ 12
7.4 Recognition and measurement: lessee ................................................................ 14
7.4.1 Recognition exemptions ....................................................................................... 14
7.4.2 Initial recognition and measurement of leases: lessee ........................................ 18
7.4.3 Interest rate implicit in the lease ......................................................................... 26
7.4.4 Subsequent measurement of leases: lessee ........................................................ 29
7.4.5 Reassessment of lease liability: lessee ................................................................. 32
7.5 Presentation and disclosure: lessee ..................................................................... 36
7.6 Comprehensive examples: lessee ........................................................................ 38

2
Learning outcomes (IFRS 16)

Learners should be able to account for and disclose leases in the financial
statements of lessees (only) in accordance with the International Financial
Reporting Standards (IFRS).

Assessment criteria
After having studied this learning unit, you should be able to
 recognise a right-of-use asset and a lease liability for all leases at the
commencement of the lease or elect not to apply this requirement for short-
term leases and leases for which the underlying asset is of low value
 initially measure a right-of-use asset and the lease liability on a present value
basis
 calculate the amortisation table using the interest rate implicit in the lease
 subsequently measure the right-of-use asset and lease liability
 record, present and disclose a lease in the financial statements of the lessee
according to the requirements of the International Financial Reporting
Standards

Overview

This learning unit will be discussed under the following sections:


7.1 Schematic representation of IFRS 16
7.2 Reporting Framework (IFRS & IFRS for Small and medium size enterprises)
7.3 Identifying a lease
7.3.1 Steps to follow to determine whether a contract contains a lease
7.3.2 Separating components of a contract
7.3.3 Lease term
7.4 Recognition and measurement: lessee
7.4.1 Recognition exemptions
7.4.2 Initial recognition and measurement of leases: lessee
7.4.3 Interest rate implicit in the lease
7.4.4 Subsequent measurement of leases: lessee
7.4.5 Reassessment of lease liability: lessee
7.5 Presentation and disclosure: lessee
7.6 Comprehensive examples: lessee

3
7.1 SCHEMATIC REPRESENTATION OF IFRS 16

STUDY

Objective

• To ensure that lessees provide relevant information in a manner that faithfully represents
those transactions.

Definition

• A lease is a contract, or part of a contract that conveys the right to use an asset (the
underlying asset) for a period of time (lease term) in exchange for consideration.

Identifying a lease

• Assess at inception of a contract whether the contract is, or contains, a lease.


• A contract is, or contains a lease if the contract conveys the right to control the use of an
identified asset for a period of time in exchange for consideration, meaning that the customer
has both of the following:
– the right to obtain substantially all of the economic benefits from use of the identified
asset
– the right to direct the use of the identified asset
Accounting by lessee

• Single lessee accounting model


• Recognise a right-of-use asset and a lease liability for all leases at the commencement of the
lease or elect not to apply this requirement for short-term leases and leases for which the
underlying asset is of low value.
• Initially measure a right-of-use asset and the lease liability on a present value basis.
• Include initial direct costs, lease payments made at or before the commencement date, less
any lease incentives received, and estimates of costs to be incurred by the lessee in
dismantling and removing the underlying asset or restoring the site on which it is located, in
the carrying amount of the right-of-use asset.

The lease payments shall be discounted over the lease term using the interest rate implicit in the
lease, if that rate can be readily determined. If this rate cannot be readily determined, the lessee
shall determine and use its own incremental borrowing rate.
• Subsequently measure a right-of-use asset similarly to other non-financial assets.
• Subsequently measure the lease liability similarly to other financial liabilities.

4
7.2 REPORTING FRAMEWORK

IFRS for SMEs Section 20 prescribes the accounting treatment (recognition and measurement) for
leases and related disclosure requirements.

Here are the main differences between IFRS 16 and Section 20 regarding the accounting treatment
for leases. See below Table 1

Table 1: Main differences between IFRS 16 / Section 20 – Leases


Main difference IFRS 16 Section 20 (IFRS for SMEs)
Scope IFRS 16 applies to all leases other Applies to leases other than the
than: following:
• Those to explore for or use • Leases to explore for or use
minerals, non-regenerative minerals, oil, natural gas and
resources. similar non-regenerative
• Licensing agreements for such resources.
items as motion picture films, • Licensing agreements for such
video recordings, plays, items as motion picture films,
manuscripts, patents and video recordings, plays,
copyrights. manuscripts, patents and
• Measurement of property held by copyrights.
lessees for investment property • Measurement of property held
and by lessors under operating by lessees for investment
leases. property and by lessors under
• Measurement of biological assets operating leases.
by lessees under finance leases • Measurement of biological assets
and by lessors under operating by lessees under finance leases
leases. and by lessors under operating
leases.
• Leases that could lead to a loss as
a result of contractual terms
unrelated of contractual terms
unrelated to changes in the price
of the leased asset, changes in
foreign exchange rates or default
by one of the counterparties.
• Onerous operating leases.
Short-term leases Operating lease payments are Operating leases are expensed on a
expensed on a straight-line basis straight-line basis or another basis
over the lease term unless another that represents the use of the asset,
systematic basis is more unless payments to the lessor
representative of the use of the increase with expected inflation in
asset. which case the payments are
expensed when payable.

5
Initial measurement No difference No difference
Subsequent No difference No difference
measurement
Disclosure No difference No difference

This learning unit is based on IFRS as the accounting framework and use IFRS 16 accounting
principles.

7.3 IDENTIFYING A LEASE

STUDY

IFRS 16 defines a lease as a contract, or part of a contract, that conveys the right to control the use
of an asset (the underlying asset) for a period of time in exchange for consideration.

An underlying asset is an asset that is the subject of a llease, for which the right to use that asset
has been provided by a lessor to a lessee.

A lessee is an entity that obtains the right to use an underlying asset for a period of time in exchange
for consideration.

A lessor is an entity that provides the right to use an underlying asset for a period of time in exchange
for consideration.

IFRS 16 Leases will be applied if a contract contains a lease. Therefore, at inception of a contract, an
entity will need to determine whether a contract is or contains a lease. As per the lease definition,
a contract is or will contain a lease when the contract conveys:
• the right to control the use of an identified asset
• for a period of time
• in exchange for consideration

A contract conveys the right to control the use of an identified asset for a period of time when the
customer (lessee) has both of the following:
• the right to obtain substantially all economic benefits from the identified asset
• the right to direct the use of the identified asset

If a customer only has the right to control the use of an asset for a portion of the term of the
contract, only that portion of the contract will contain a lease.

A period of time refers to the time or amount of use of an identified asset (e.g. the number of
production units that the equipment will be used to produce).

An entity is only required to reassess whether a contract is or contains a lease when the terms or
conditions of the lease have changed.

6
7.3.1 Steps to follow to determine whether a contract contains a lease. Source: IFRS 16.B31

EXAMPLE 1: Does a contract contain a lease?

Sunshine Ltd (lessee) entered into a contract with Energex Ltd (lessor) to rent property space of
20m2 of Energex Ltd’s 100 m2 property to sell its solar panel for a period of three years. Energex Ltd
has the right to change the location allocated to Sunshine Ltd at any time during the period of use.
There are minimal costs to Energex Ltd associated with changing the space used by Sunshine Ltd.
Sunshine Ltd makes use of a mobile truck (which it owns) to sell solar panels that can be easily
moved. Sunshine Ltd will pay Energex Ltd R8 500 per month to make use of the property space.

REQUIRED

7
SOLUTION 1

As per IFRS 16 Leases, a contract contains a lease when the contract conveys the following:
• the right to control the use of an identified asset
• for a period of time
• in exchange for consideration

Discussion of lease definition components:

Right to control the use of an identified asset


As per IFRS 16 Leases, a contract conveys the right to control the use of an identified asset when the
customer (lessee) has both of the following:
• the right to obtain substantially all economic benefits from the identified asset
• the right to direct the use of the identified asset

The contract specifies the amount of property space that Sunshine Ltd is entitled to use of the entire
property space owned by Energex Ltd. However, the property space is not fixed and as a result there
is no identified asset.

Even though Sunshine Ltd controls its owned mobile truck, the contract is for property space on
Energex Ltd’s property and has the right to change the property space allocated at any time.
Sunshine Ltd does not control the asset because:
• Sunshine Ltd does not have substantially all the economic benefits from the property space.
Energex Ltd could benefit economically from substituting the property space as there are
minimum costs involved in moving the mobile truck used by Sunshine Ltd. Energex Ltd
benefits from substituting the property space at its own property for something else that will
be most effective in increasing its own economic benefits.
• Energex Ltd has the right and practical ability to change the property space used by
Sunshine Ltd anytime during the entire period, thereby directing the use.

For a period of time


• The contract is for a three-year period.

In exchange for consideration


• Sunshine Ltd will pay Energex Ltd an amount of R8 500 per month.

Conclusion:

There is no identified asset and Sunshine Ltd does not have the right to control an identified asset.
Not all the conditions have been met and as a result, the contract does not contain a lease.

8
EXAMPLE 2: Does the contract contain a lease?

Boat Ltd (customer) is a manufacturer of luxury motorboats. On 1 June 20.12, Boat Ltd (lessee)
signed a long-term agreement with Part Ltd (supplier). In terms of the agreement, Part Ltd must
supply portholes to Boat Ltd daily for a period of ten years. To ensure that Part Ltd fulfils its
obligations in terms of the agreement, the management of Part Ltd decided to construct a plant for
manufacturing portholes on the premises of Boat Ltd. The close proximity of the plant will ensure
that portholes can be supplied as and when needed by Boat Ltd.

Other relevant matters:


• To ensure the continuous availability of portholes, the arrangement stipulates that Boat Ltd
must pay an annual fixed charge of R150 000 (at the end of the year) to Part Ltd.
• Since the portholes are unique, the plant has no other purpose and will be demolished at the
end of the agreement.
• The purchases by Boat Ltd for the first year of production amounted to 500 portholes at
R2 500 per unit – not yet transferred to work-in-progress.
• The incremental borrowing rate of Boat Ltd is 11,5% per annum.

REQUIRED

Discuss whether the contract between Boat Ltd and Part Ltd contains a lease.

SOLUTION 2

As per IFRS 16 Leases, a contract contains a lease when the contract conveys the following:
• a right to control the use of an identified asset
• for a period of time
• in exchange for consideration

Discussion of lease definition components:

Right to control the use of an identified asset


As per IFRS 16 Leases, a contract conveys the right to control the use of an identified asset when the
customer (lessee) has both of the following:
• the right to obtain substantially all economic benefits from the identified asset
• the right to direct the use of the identified asset

There is an identified asset:


• Although the portholes manufacturing plant is not mentioned in the long-term agreement
between Boat Ltd and Part Ltd, this asset is implicitly specified, because for Part Ltd to fulfil
the requirements of the agreement, it must erect the plant. It is also unlikely that Part Ltd
can/would substitute the plant with an alternative plant. Part Ltd would also not benefit
economically from substituting the asset – the costs associated with substituting the plant
would be too high.
9
• Boat Ltd has the right to obtain all the economic benefits from the plant – the portholes are
unique and Boat Ltd is the only user thereof.
• Boat Ltd has the right to direct how and for what purposes the plant is used throughout the
period of use – Boat Ltd has the decision-making rights to, for example, change the type of
portholes that is produced, whether portholes are produced and how many are produced.

Boat Ltd has the right to control the use of the plant throughout the ten-year period of use because:
• The plant is an identified asset, a manufacturing plant.
• Boat Ltd has the right to obtain substantially all the economic benefits from the use of the
manufacturing plant over the ten-year period of use. Boat Ltd has exclusive use of the plant
throughout the period of use.
• Within the scope of its right of use defined in the contract, Boat Ltd makes the relevant
decisions about how and for what purpose portholes are used by being able to decide, for
example, the change of type of portholes, whether portholes are being produced and how
many. Boat Ltd has the right to change these decisions during the ten-year period of use.

Therefore, Boat Ltd has the right to benefit economically from the manufacturing plant and has the
right to direct the use thereof.

For a period of time


• The contract is for a ten-year period.

In exchange for consideration


• Boat Ltd will pay Part Ltd an annual fixed amount of R150 000 at the end of the year.

Conclusion:
All the conditions of the lease definition have been met. The arrangement coveys the right to
Boat Ltd to control the use of the plant and it can be concluded that the arrangement contains a
lease.

7.3.2 Separating components of a contract

STUDY
Contracts often combine different kinds of obligations of the supplier, which
might be a combination of lease components or a combination of lease and
non-lease components. If such a multi-element arrangement exists, each separate
lease component should be identified (using the guidance on the definition of a lease) and be
accounted for separately from non-lease components, unless the entity applies the practical
expedient. As a practical expedient, lessees don’t have to separate non-lease components from
lease components, and instead account for each lease component and any associated non-lease
components as a single lease component.

Each lease component within a contract that is, or contains, a lease must be accounted for as a lease
separately from the non-lease components of the contract. This does not apply to entities applying
the practical expedient. As a practical expedient, lessees don’t have to separate non-lease
components from lease components, and instead account for each lease component and any
associated non-lease components as a single lease component.
10
The right to use an underlying asset is a separate lease component if both of the following criteria
are met:
• The lessee can benefit from use of the underlying asset either on its own or together with
other resources that are readily available (goods and services that are sold or leased
separately or resources that the lessee has already obtained) to the lessee.
• The underlying asset is neither highly dependent on, nor highly interrelated with, the other
underlying asset in the contract.

The lessee may elect (as a practical expedient), by class of underlying asset, not to separate the non-
lease components from the lease component.

If the lessee could, for example, decide not to lease a specific underlying asset without significantly
affecting its rights to use other underlying assets in the contract, it might indicate that the specific
underlying asset is not highly dependent on, or highly interrelated with, the other underlying assets.

If there are separate lease and non-lease components in the contract, the lessee shall allocate the
consideration in the contract to each lease component on the basis of its relative stand-alone price.
The relative stand-alone price of the lease and non-lease components shall be determined on the
basis of the price the lessor, or a similar supplier, would charge an entity for only that component,
or a similar component, separately. If observable stand-alone prices are not readily available, the
lessee shall estimate the prices by maximising the use of observable information.

Components of contract

Lessee does not elect practical Lessee elects practical expedient.


expedient.

For contracts that contain a lease component The lessee accounts for lease and non-
and one or more additional lease or non-lease lease components as a single lease
components, the lessee is required to: component.
• Allocate the consideration in the contract to • Lease liability will increase with the non-
each lease component on the basis of the lease components (which will result in an
relative stand-alone price of the non-lease increase in the right-of-use-asset).
components
• Account for non-lease components applying
other applicable standards.

11
Relative stand-alone prices
The relative stand-alone price of lease and non-lease components is determined
on the basis of the price the lessor, or a similar supplier, would charge an entity
for that component, or a similar component.

IF an observable stand-alone price is not available, the lessee will estimate the
stand- alone price, maximising the use of observable information.

EXAMPLE 3: ALLOCATION OF CONSIDERATION TO LEASE AND NON-LEASE COMPONENTS

T Ltd entered into a lease contract in terms of which it will lease a bus. The contract is a lease in
terms of IFRS 16. The total consideration payable under the contract is R200 000 per annum for
three years. T Ltd determines that the contract consists of two separate components namely, the
lease of the bus and the maintenance of the bus.

According to the accounting policy of T Ltd, lease components and non-lease components should
be separated. T Ltd establishes that similar maintenance services are provided by third parties at
R25 000 per year and the relative stand-alone rental amount for a similar bus is R180 000 per
annum.

SOLUTION 3:
The annual consideration of R200 000 will therefore be allocated as follows:
R
Bus (R180 000 / R205 000 (R180 000 + R25 000) × R200 000) 175 610
Maintenance (R25 000 / R205 000 (R180 000 + R25 000) × R200 000) 24 390
200 000
7.3.3 Lease term

STUDY
IFRS 16 defines lease term as the non-cancellable period of the lease for which the
lessee has the right to use the underlying asset, as well as periods covered by an
option to extend or an option to terminate if the lessee is reasonably certain to
exercise the extension option or not exercise the termination option.

An entity will determine the lease term as the non-cancellable period of a lease together with both
the:
• period covered by an option to extend the lease if it is reasonably certain, the lessee will
exercise that option; and
• periods covered by an option to terminate the lease if it is reasonably certain, the lessee will
not exercise that option.

The lease term begins at the commencement date and includes any rent-free periods provided to
the lessee by the lessor. The commencement date of the lease is the date on which a lessor makes
an underlying asset available for use by a lessee.

12
In determining the length of the non-cancellable period of the lease, an entity shall apply the
definition of a contract and determine the period for which the contract is enforceable. A lease is
no longer enforceable when the lessee and the lessor each have the right to terminate the lease
without permission from the other party with no more than an insignificant penalty. If only a lessee
has the right to terminate a lease, that right is considered to be an option to terminate the lease
available to the lessee which an entity considers when determining the lease term. If only a lessor
has the right to terminate a lease, the non-cancellable period of the lease includes the period
covered by the option to terminate the lease.

In assessing whether a lessee is reasonably certain to exercise, or not to exercise, the option to
extend or terminate, all relevant facts and circumstances that create an economic incentive for the
lessee to exercise, or not to exercise the option, must be considered. A lessee’s past practice may
also provide helpful information in assessing whether the lessee is reasonably certain to exercise,
or not to exercise, an option.

EXAMPLE 4: IDENTIFYING THE LEASE TERM

E Ltd (lessee) entered into the following lease agreements:

Lease A: E Ltd has the right to use a machine for three years. The first year is rent free and R100 000
is payable at the end of year 2 and year 3. The lease term is three years. Even though the
first year is rent free, E Ltd has the right to use the machine during the first year (total of
three years).

Lease B: E Ltd has the right to use a vehicle for three years, and has the option to extend the lease
for another two years. At the commencement date of the lease, E Ltd is not reasonably
certain to exercise the option to extend the lease. The lease term is three years.

Lease C: E Ltd has the right to use a delivery truck for three years and has the option to extend the
lease for another two years. E Ltd modifies the delivery truck to meet its specific needs.
At the commencement date of the lease, E Ltd is reasonably certain to exercise the option
to extend the lease. The lease term is five years.

SOLUTION 4

COMMENTS:

➢ Judgement may be needed to assess whether an entity is “reasonably certain” to exercise any
option attached to a lease.
➢ An entity need to account for a reassessment of the lease liability (refer to section 6.6 for more
detail) should the lease term be revised (for example, if during the lease term, it becomes
reasonably certain that the option to extend the lease will be exercised where this was not
reasonably certain at the commencement of the lease; or the extension of the lease is no longer
reasonably certain).

13
7.4 RECOGNITION AND MEASUREMENT: LESSEE

STUDY
Legally, the lessee is not the owner of the leased asset and is not required to take
ownership of the leased asset at the end of the lease term. However, the
substance of the agreement and its financial reality is that the lessee obtains the
right to use the asset to generate economic benefits for itself over the lease term.

For this reason, the lessee is required to recognise both:


• an asset (right-to-use asset) and;
• a liability (lease liability);
on its statement of financial position for all assets leased by it under lease
agreements.

7.4.1 Recognition exemptions

A lessee may elect not to recognise underlying assets and liabilities for:
• Leases of 12 months or less (short-term leases); and
• Leases for which the underlying assets are low-value assets, for example tablets, personal
computers and small office furniture and items.

If this exemption is elected, the lease payments are recognised as an expense in the profit or loss
section of the statement of profit or loss and other comprehensive income on a straight-line basis
over the lease term, unless another systematic basis is more representative of the pattern of the
lessee’s benefit. In terms of SAICAs Circular 5/2018 Recognition of lease income and expense on a
basis other than the straight-line basis under IFRS 16 – Leases, the use of ‘another systematic basis’
is expected to be rare. When applying such other systematic basis, the pattern of the user’s benefit
is only affected by factors which impact the physical usage of the underlying asset. Where the
straight-line basis is used and cash flows are not equal, the difference between the cash flows and
the expense in the statement of profit or loss and other comprehensive income will end up in the
statement of financial position as an accrued or prepaid expense.

“Low value” = IFRS 16.BC100 states that assets, when new, with a value of $5 000 would be
considered low-value assets.

Assets that are highly dependent, highly interrelated with other assets will not qualify as low-value
assets.

For the exam


We will always state in a question if an asset is a low-value asset and whether or
not the company applies the recognition exemption to its low-value and short-
term assets.

14
7.4.1.1 Short-term leases

An option to extend or the terminate a lease that is reasonably certain to be exercised should be
considered when determining whether the lease term is 12 months or less. A lease that contains a
purchase option is not a short-term lease.

When the recognition exemption is considered, low-value assets should elect the exemption on a
lease-by-lease basis and short-term assets by the class of the underlying assets.

7.4.1.2 Low-value underlying assets

An underlying asset can also only be of low value if:


• The lessee can benefit from the use of the underlying asset on its own or in combination with
other resources that are readily available to the lessee; and
• The underlying asset is not highly dependent on, or highly interrelated with, other assets.

When the recognition exemption is elected, the accounting treatment for these specific short-term
and low-value assets are as follows:

Recognition of lease payments

The lessee will recognise the lease payments associated with leases as an expense on:
• Straight-line basis over the lease term; or
• Another systematic basis if it is more representative of the pattern of the lessee’s benefit.

Where the straight-line basis is used and cash flows are not equal, the difference between the cash
flows and the expense in the statement of profit or loss and other comprehensive income will end
up in the statement of financial position as an accrued or prepaid expense.

EXAMPLE 5: LEASES OF LOW VALUE

Zumba Ltd (lessee) provides training and online professional development courses. Zumba Ltd has
the following leases that have non-cancellable terms in excess of 12 months:
▪ lease of its office building;
▪ leases of office furniture such as boardroom tables, chairs and couches;
▪ leases of company cars; and
▪ leases of numerous items of IT equipment, such as laptops and data projectors.

SOLUTION 5:

Zumba Ltd determines that the leases of its office furniture and IT equipment qualify for the
recognition exemption in IFRS 16 on the basis that these underlying assets, when they are new, are
individually of low value. Consequently, the lease payments will be recognised as an annual expense
on a straight-line basis in profit or loss. Zumba Ltd will apply the recognition and measurement
principles of IFRS 16 for the leases relating to the office building and company cars and would
recognise a right-of-use asset and a lease liability.

15
Disclosure

• A lessee must disclose the following amounts in a tabular format for the reporting period, unless
another format is more appropriate:
– The expense relating to the short-term lease. This expense may exclude leases with a lease
term of one month or less;
– The expense relating to leases of low-value assets, excluding expenses relating to short-term
leases included above;
– Total cash outflow for leases.
• A lessee shall disclose additional qualitative and quantitative information about its leasing
activities necessary to give users of the Annual Financial Statements a basis to assess the effect
of the lease on the lessee.
• A lessee that accounts for short-term leases or leases of low-value assets applying the exemption
criteria of 7.4.1 must state that fact.
• A lessee is required to disclose the amount of its lease commitments for short-term leases if the
portfolio of short-term leases to which it is committed at the end of the reporting period is
different to the portfolio of short-term leases to which the short- term lease expense is already
disclosed in terms of the disclosure above.

EXAMPLE 6: ACCOUNTING FOR A LEASE FOR WHICH THE UNDERLYING ASSETS ARE OF LOW
VALUE

The end of the reporting period of Z Ltd is 31 December 20.29. Zet Ltd entered into a non-
cancellable lease on 1 January 20.29 to lease five laptop computers for its employees from R Ltd.
The contract is a lease in terms of IFRS 16.

The following information is applicable to the lease contract:

The initial lease term is six years. The lease payments are R2 500 per month for the first four years
and R1 500 per month for the final two years. Z Ltd has the option to extend the lease term for a
further two years at R1 000 per month. At commencement of the lease Z Ltd is reasonably certain
that it will exercise the option to extend the lease term by a further two years.

10% of every lease payment goes towards covering the maintenance costs incurred by and is to be
paid for by Rent Limited. These values are in line with costs for similar maintenance services
rendered by third parties.

Z Ltd elected to apply the recognition exemption in respect of low value assets to this lease
agreement (IFRS 16.5). Z Ltd accounts for the lease and the non-lease components separately
(IFRS 16.12).

16
SOLUTION 6:

Calculation of the straight-line amount of the lease

Total amount actually paid or payable R


Years 1–4 R2 500 × 48 months 120 000
Years 5–6 R1 500 × 24 months 36 000
Years 7–8 R1 000 × 24 months 24 000
180 000

Amount in respect of maintenance (R180 000 × 10%) (18 000)


Lease component 162 000
Lease term 8 years
Annual lease expense (R162 000/8 years) 20 250

Journal entries
Dr Cr
R R
Years 1–4
Maintenance (P/L) (30 000 × 10%) 3 000
Lease expense (P/L) 20 250
Prepayments (balancing) (SFP) 6 750
Bank (SFP) (2 500 × 12) 30 000
Recognition of straight-line lease expense for low value assets

Years 5–6
Maintenance (P/L) (18 000 × 10%) 1 800
Lease expense (P/’L) 20 250
Prepayments (balancing) (SFP) 4 050
Bank (SFP) (1 500 × 12) 18 000
Recognition of straight-line lease expense for low value assets

Years 7–8
Maintenance (P/L) (12 000 × 10%) 1 200
Lease expense (P/L) 20 250
Prepayments (balancing) (SFP) 9 450
Bank (SFP) (1 000 × 12) 12 000
Recognition of straight-line lease expense for low value assets

Comments:

➢ A similar approach would be followed for short-term leases where the recognition exemption was
elected. There would only be a prepaid/accrued amount in the statement of financial position if the
payments are not equal to the lease expense (straight-line) and the lease term (< 12 months) is
during two financial periods.

17
Disclosure Requirements (recognition exemption)

The disclosure of the information provided above of Z Ltd (Example 6) will be as follows:

Z Ltd
Notes for the year ended 31 December 20.30

28. Lease agreements in which the company is a lessee


28.1 Income and expenses related to leases
20.30 20.29
R R
Expenses
Maintenance expense 3 000 3 000
Low-value assets lease expense 20 250 20 250
The company has elected to apply the simplified accounting method for its low value lease of laptop
computers.

Comments

➢ The prepayment of R6 750 in 20.29 and R13 500 (cumulative balance for R6 750 over two years)
in 20.30 is presented in the statement of financial position under non-current assets – Pre-
payments.

7.4.2 Initial recognition and measurement of leases: Lessee

STUDY

Initial recognition and measurement of right of use asset

The Conceptual Framework for Financial Reporting (Conceptual Framework) defines an asset as a
present economic resource, which is a right that has the potential to produce economic benefits,
controlled by the entity as a result of past events. In terms of a lease agreement, a lessee would
have a right to use an underlying asset for the lease term as the use of the asset is under its control
(legally established under the agreement). An entity controls an economic resource if it has the
present ability to direct the use of it and obtain the economic benefits that may flow from it.
Consequently, the lessee should recognise a right-of-use asset when entering into a lease
agreement, unless the recognition exemption (section 7.4.1 above) is elected. At the
commencement date, the right-of-use asset is measured at cost.

18
At commencement of the lease, a lessee shall recognise a right-of-use asset (e.g. machinery) and a
lease liability (future lease instalments payable) in its statement of financial position.

LESSEES

RIGHT-OF-USE OF ASSET LEASE LIABILITY

Initial recognition: Initial recognition:

At commencement date, the lessee will At commencement date, the lessee will
recognise the right-of-use asset at cost. recognise the lease liability at the present value
(using the interest rate implicit in the lease of
the lease payments not paid at that date).

Cost comprises: Lease payments used to measure the lease


liability comprise:

• the amount at which the lease liability is • fixed payments (includes in-substance fixed
initially measured payments) less lease incentives receivable
• any lease payments made at or before the • variable lease payments that depend on an
commencement date, excluding any lease index or a specific rate, initially to be
incentives received measured using the index or rate as at
• initial direct costs incurred by the lessee commencement date
• an estimate of the costs that will be • amounts payable in terms of residual value
incurred by the lessee to dismantle or guarantees
remove the underlying asset, restoring the • exercise price of a purchase option if it is
site on which the asset is located or reasonably certain that the lessee will
restoring the underlying asset to the exercise that option
condition required in terms of the lease, • penalty payments for terminating the lease
unless those costs are incurred to produce
inventory

EXAMPLE 7 INITIAL MEASUREMENT OF RIGHT-OF-USE ASSET

T Ltd (lessee) leases a machine under a lease agreement from 1 June 20.12 from K Ltd (lessor). T Ltd
did not elect the simplified accounting treatment for the machine.

The details of the lease agreement are as follows:

Lease term 3 years


Payment made on 27 May 20.12 relating to the design of the machine R19 500
Non-refundable deposit paid on 26 May 20.12 to secure the lease R15 000
Legal fee paid to a legal adviser to check the contract R2 500
50% of the legal fee reimbursed by Tembe Ltd in cash R1 250
Cost to assemble the machine R5 000

19
Annual inspection cost to be paid by Thabo Ltd R3 500
Estimated future dismantling cost to be paid on 31 May 20.15 R7 000
Pre-tax discount rate applicable to the dismantling provision 9%

Initial measurement of lease liability on 1 June 20.12 (being the present R46 000 value of future
lease payments).

SOLUTION 7:

On commencement date, T Ltd will recognize a right-of-use asset for the use of the machine at the
following amount:

R
Lease liability 46 000
Lease payments made before the commencement date 15 000
Initial direct costs (2 500 + 5 000) 7 500
Less: lease incentive received (1 250)
Inspection cost -
Cost relating to the design of the machine -
Dismantling cost (FV = 7 000; n = 3; i = 9%; PV = ?) 5 405
Total cost of right-of-use asset 72 655

Comments:

➢ The non-refundable lease payment paid in advance to secure the lease will not form part of the
present value of the lease liability (it has already been paid and is not part of the liability to pay
the future lease payments). Only lease payments that are not paid at commencement date will
be included in the initial measurement amount of the lease liability. The deposit is still included
in the initial cost of the right-of-use asset.
➢ The lessee shall apply IAS 16 to account for inspection cost when paid in future. The inspection
cost does not relate to the lease and the right to use the machine but is a cost of actually using
it.
➢ If a lessee incurs costs relating to the construction or design of an underlying asset, the lessee
shall account for those costs when applying other applicable Standards, such as IAS 16
(IFRS 16.B44). Such costs would typically be treated as a leasehold improvement and be
recognised as an item of property, plant and equipment. Costs relating to the construction and
design of the underlying asset are not incurred in connection with the right to use the
underlying asset.
➢ Since the dismantling costs do not arise from the production of inventories, it is capitalised to
the right-of-use asset.

20
JOURNAL ENTRIES
Dr Cr
May and June 20.12 R R
Right-of-use asset (SFP) 46 000
Lease liability (at PV) (SFP) 46 000
Right-of-use asset (SFP) 15 000
Bank (SFP) 15 000
Right of use asset (initial direct costs) (SFP) 7 500
Bank (SFP) ( 2 500 legal fees + 5 000 assembly costs) 7 500
Bank (SFP) (50% x 2 500 legal fees) 1 250
Right-of-use asset (SFP) 1 250
Right-of-use asset (SFP) 5 405
Dismantling provision (SFP) 5 405
Initial recognition of lease, initial direct costs and lease incentive received
Property, plant and equipment (SFP) 19 500
Bank (SFP) 19 500
Recognition of design cost as PPE

Initial recognition and measurement of lease liability:


The Conceptual Framework defines a liability as a present obligation of the entity to transfer an
economic resource as a result of past events. In terms of a lease agreement, a lessee would have
the obligation to make certain cash payments (transfer of an economic resource) and would have
no practical ability to avoid making such payments as it is legally bound by the lease contract (legal
obligation). Consequently, at the commencement date of the lease, the lease should recognised the
lease liability, unless the recognition exemption (section 6.1 above) is elected.

At the commencement date, a lessee shall measure the lease liability at the present value of the
lease payments that are not paid at that date.

The lease payments shall be discounted over the lease term using the interest rate implicit in the
lease, if that rate can be readily determined (the interest rate implicit in the lease is determined
from the perspective of the lessor). If that rate cannot be readily determined, the lessee shall use
its incremental borrowing rate.

In-substance fixed payments are lease payments that, in form, contain variability but, in substance,
are fixed, for example, where payments must be made if the asset is proven to be capable of
operating during the lease term, or where payments must be made only if an event occurs that has
no genuine possibility of not occurring.

21
EXAMPLE 8: IN SUBSTANCE FIXED PAYMENTS

F Ltd provides various training classes, including yoga. F Ltd leases a studio from Health Ltd in terms
of a lease contract. The lease contract specifies that F Ltd must pay an amount of R400 per hour for
the use of the studio with a minimum annual payment of R500 000. F Ltd expects to use the studio
for 1 500 hours per year.

▪ The R500 000 represents an in-substance fixed payment per year.


▪ F Ltd will pay an annual amount of R600 000 (R400 × 1 500). The difference between the R600 000
and the fixed annual payment of R500 000 is a variable payment. This variable payment is not
dependent on an index or a rate.

Consequently, the R500 000 in-substance fixed payment will be included in the initial measurement
of the lease liability and the R100 000 variable payment will be included as an expense in profit or
loss.

Variable lease payments that depend on an index or rate are unavoidable, because uncertainty
relates only to the measurement of the amount, but not to its existence; consequently, they form
part of the lease liability for the lessee/net investment for the lessor. Such variable lease payments
are initially measured using the index or the rate at the commencement date. The entity does not
forecast future changes of the index/rate; changes are only taken into account at the point in time
at which the lease payments actually changes

EXAMPLE 9: LEASE PAYMENTS THAT DEPEND ON AN INDEX

M Ltd (lessee) operates in an inflationary environment. On 1 March 20.16, M Ltd entered into a six-
year lease contract with annual lease payments of R250 000, payable at the beginning of each year.
Every two years, lease payments will be adjusted to reflect changes in the Consumer Price Index
(CPI) for the preceding 24 months. On 1 March 20.16, the CPI was 125.

On 1 March 20.16, the lease liability is calculated based on the lease payments of R250 000 per
year. M Ltd will only remeasure the lease liability on 1 March 20.18, when the contractual cash flows
actually change based on the CPI on that date.

Variable lease payments based on the future amount of something that changes other than with
the passage of time or not based on an index or rate (e.g. a lease payment linked to a lessee’s
performance derived from the underlying asset, such as payments of a specified percentage of sales)
are not part of the lease liability. Such lease payments, often referred to as “contingent lease
payments”, are recognised in profit or loss (expenses) in the period in which the event or condition
that triggers such payments occurs.

22
EXAMPLE 10: VARIABLE LEASE PAYMENTS LINKED TO SALES

Assume the same facts as Example 9 above, except that M Ltd is also required to make variable lease
payments for each year of the lease, which are determined as 2,5% of M Ltd’s audited sales
generated from the underlying asset.

At the commencement date, the lease liability will be recognised at the same amounts as in
Example 9. This is because the additional variable lease payments are linked to future sales and,
thus do not meet the definition of lease payments. Consequently, such contingent payments are
not included in the measurement of the lease liability.

If M Ltd’s audited sales generated from the underlying asset for the first year of the lease are
R1 000 000, M Ltd will recognise an expense (P/L) of R25 000 (R1 000 000 × 2,5%) in its statement
of profit or loss and other comprehensive income for the year ended 28 February 20.17. The contra
entry (credit) to this journal entry will probably be recorded as an accrual at year end, because
M Ltd’s sales generated from the underlying asset will first need to be audited after year end to
determine the exact amount.

Lease agreements often include a residual value for the underlying asset. It was also mentioned
above that the amount expected to be paid under a residual value guarantee, is included in the
initial measurement of the lease liability for the lessee.

The theory is that the residual value will be equal to (or less than) the estimated market value of the
asset at the end of the lease term. This will enable the lessor to sell the asset for that amount. The
residual value will be an estimate and the lessor will not be certain what the market value of the
asset will be at the end of the lease term. However, at the commencement date of the lease, the
lessee and the lessor can have a contractual agreement where they agree on a residual value
guarantee amount of the underlying asset at the end of the lease term. The residual value amounts
can be fixed (i.e. a contractually agreed amount that will be paid regardless of the market value of
the underlying asset) or variable (i.e. expected selling price of the asset in the open market). A
residual value guarantee will be variable if, for example, the estimated market value of the asset is
lower than the agreed residual value guarantee then the lessee will have to pay the difference to
the lessor. This amount is the expected amount payable by the lessee under residual value
guarantees.

The standard defines an unguaranteed residual value as that portion of the residual value of the
underlying asset, the realisation of which by a lessor is not assured or is guaranteed solely by a party
related to the lessor. When an unguaranteed residual value is attached to a lease, the asset will
normally be returned to the lessor at the end of the lease term.

23
EXAMPLE 11: INTIAL MEASUREMENT OF LEASE LIABILITY

The end of the reporting period of P Ltd is 31 December. On 1 January 20.16 P Ltd entered into a
lease agreement with R Ltd to lease a new office building from R Ltd for a non-cancellable period of
ten years, starting on 1 January 20.16. At the end of the lease term, P Ltd has guaranteed R Ltd that
they will receive a residual value of at least R25 000 000 for the office building. On 1 January 20.16,
P Ltd expects that it will have to make a payment of R2 000 000 under the residual value guarantee
(i.e. it expects that the market value of the office building will be R23 000 000 and P Ltd need to pay
the shortfall of R2 000 000). P Ltd does not have sufficient information to determine the interest
rate implicit in the lease. The incremental borrowing rate of P Ltd is 12%.

The following information has been extracted from the lease contract:

▪ 1 January 20.16 is the commencement date of the lease;


▪ Deposit of R500 000 paid on 15 December 20.15 to secure the lease;
▪ P Ltd incurred legal fees of R20 000 relating to this lease contract. R Ltd partially reimbursed
P Ltd and paid R10 000 over to P Ltd on 1 January 20.16; and
▪ Annual lease payments, payable in arrears, are R2 500 000.

P Ltd should initially recognise the lease liability at the present value of the unpaid lease payments,
using the incremental borrowing rate of 12%, which is R14 769 504 (FV=2 000 000, N=10,
PMT=2 500 000, I = 12%).

SOLUTION 11:

The journal entries for the initial recognition of the right-of-use asset and the lease liability will
therefore be as follows:
Dr Cr
R R
Journal entries:

15 December 20.15
Lease deposit debtor (SFP) 500 000
Bank (SFP) 500 000
Pay deposit on lease before commencement date

1 January 20.16
Right-of-use asset (SFP) 20 000
Bank (SFP) 20 000
Capitalise initial direct costs

Bank (SFP) 10 000


Right-of-use asset (SFP) 10 000
Lease incentive paid by lessor

24
Right-of-use asset (SFP 15 269 504
Lease liability (SFP) 14 769 504
Lease deposit debtor (SFP) 500 000
Recognise right-of-use asset and lease liability

Comment:

➢ Lease incentives which have been received before or on commencement date are deducted
from the initial measurement of the right-of-use asset. Lease incentives not yet received at
commencement date reduce the initial measurement of the lease liability.

Lease incentives receivable:

Use the same information as above, but assume the initial direct costs that Platinum Ltd agreed to
reimburse will not be paid in cash. Instead Peglarea Ltd can reduce its first lease instalment with
that amount.

The present value of the lease incentive receivable on initial recognition is R8 929 (FV=10 000, N=1,
I=12%, PMT=0)

The journal entries for the initial recognition of the right-of-use asset and the lease liability will
therefore be as follows:
Dr Cr
R R
Journal entries:

15 December 20.15
Lease deposit debtor (SFP) 500 000
Bank (SFP) 500 000
Pay deposit on lease before commencement date

1 January 20.16
Right-of-use asset (SFP) 20 000
Bank (SFP) 20 000
Capitalise initial direct costs

Right-of-use asset (SFP 15 260 575


Lease liability (SFP) (14 769 504 – 8 929) 14 760 575
Lease deposit debtor (SFP) 500 000
Recognise right-of-use asset and lease liability

25
7.4.3 Interest rate implicit in the lease

STUDY
The interest rate implicit in the lease is calculated from the perspective of the
lessor and therefore takes the unguaranteed residual value into account.

The interest rate implicit in the lease is the rate of interest that causes the present value of:
• the lease payments; and
• the unguaranteed residual value
to equal the sum of the FV of the underlying asset and any initial direct costs of the lessor, for
example legal costs and commissions in negotiating and arranging a lease. Consequently, both the
guaranteed residual value and the unguaranteed residual value are taken into account when
calculating the interest rate implicit in the lease.

See annexure A (at the end of this learning unit) for examples and explanations on amortisation
tables, interest rates and present value calculations.

EXAMPLE 12: STRUCTURING A LEASE

On 1 January 20.16 S Ltd (lessor) entered into a lease agreement with K Ltd (lessee) to lease a vehicle
to K Ltd for a non-cancellable period of two years, starting on 1 January 20.16.

S Ltd bought the vehicle for R500 000 on 1 January 20.16 and paid legal fees of R20 000 relating to
this lease contract. These amounts represents the cash outflow for the lessor, which the lessor
would want to recover from the lessee through future lease payments and, where applicable, by
selling the asset at the end of the lease term. These amounts would be used as the ‘present value’
(PV) to calculate the lease payments to be made by K Ltd over the lease term of two years (‘period’
(N) of 2 would be used).

At the end of the lease term, S Ltd expects to sell the vehicle in the market for R60 000
(unguaranteed residual value). This amount represents a future cash inflow for the lessor and would
be used as the ‘future value’ (FV) to calculate the lease payments to be made by K Ltd over the lease
term.

S Ltd requires a return of 10% to recoup its investment in the lease (i.e. the net cash outflows made
in respect of the lease). S Ltd would use these inputs to determine the lease payments.

Consequently, S Ltd would require K Ltd to make two annual payments of R271 048 (PV = -520 000;
I = 10%, N = 2, FV = 60 000) in arrears to recover it initial outflow of cash.

The time line for the cash flows can be illustrated as follows:

Commence date End of year 1 End of year 2


-500 000 PMT = +271 048 PMT = +271 048
-20 000 Sale at end:
PV = -520 000 FV = +60 000

26
Discounted cash flow = R520 000

Under the ‘time value of money’ concept, the present value of the future cash flows (two payments
of R271 048 each at the end of each year, and the sale of the vehicle at the end of the lease term)
discounted at 10%, will result in a present value of R520 000. For a lease, the discount rate is referred
to the ‘interest rate implicit in the lease’ (here 10% per year) (being the rate of interest that causes
the present value of the lease payments (R271 048) and the unguaranteed residual value (R60 000)
to equal the sum of the fair value of the underlying asset (R500 000) and any initial direct costs of
the lessor (R20 000).

Amortisation table for Springbok Ltd (lessor)

Date PMT Interest, 10 % Capital Balance


(a) (b) (c) (d)
R R R R
1 January 20.16 520 000
31 December 20.16 271 048 52 000 219 048 300 952
31 December 20.17 271 048 30 096 240 952 60 000

(a) Annual lease payment resulting in a return of 10% on the net investment.
(b) 10% on the prior balance in (d).
(c) (a) minus (b) = capital redemption on instalment.
(d) The prior balance less (c).

SOLUTION 12

The journal entries in the books of Springbok Ltd (lessor) for the initial recognition of the finance
lease will therefore be as follows:
Dr Cr
R R
Net investment in the lease (SFP) 520 000
Vehicle (SFP) 500 000
Bank (SFP) (initial direct costs) 20 000
Initial recognition of finance lease

Lessee:

K Ltd(lessee) will initially recognise the lease liability (and here the right-of-use asset as well) at the
present value of the lease payments using the above interest rate implicit in the lease of 10%, which
is R470 414 (FV = 0, N = 2, PMT = 271 048, I = 10%).

27
Amortisation table for Kudu Ltd (lessee)

Date PMT Interest, 10 % Capital Balance


(a) (b) (c) (d)
R R R R
1 January 20.16 470 414
31 December 20.16 271 048 47 041 224 007 246 407
31 December 20.17 271 048 24 641 246 407 0

Comments:

The vehicle will be returned to the lessor at the end of the lease term and the lessee is not a party
to the subsequently sale thereof by the lessor in the market. Consequently, the lessee will use a
future value of Rnil to calculate the present value of its lease liability.

The journal entries in the books of Kudu Ltd (lessee) for the initial recognition of the right-of- use
asset and the lease liability will therefore be as follows:

Dr Cr
R R
Right-of-use asset (SFP 470 414
Lease liability (SFP) 470 414
Recognise right-of-use asset and lease liability

Comment:

The subsequent treatment of a right of use asset and a lease liability is discussed in section 7.4.4
below.

EXAMPLE 13: INTEREST RATE IMPLICIT IN THE LEASE

On 1 January 20.16 P Ltd entered into a lease agreement with R Ltd to lease a new office building
from R Ltd for a non-cancellable period of ten years, starting on 1 January 20.16. At the end of the
lease term, P Ltd has guaranteed R Ltd that they will receive a residual value of at least R25 000 000
for the office building. On 1 January 20.16, P Ltd expects that it will have to make a payment of Rnil
under the residual value guarantee. R Ltd determined on 1 January 20.16 that it will be able to sell
the building to an independent third party for R28 000 000 at the end of the lease term.
Consequently, as R Ltd expects to sell the building at the end of the lease term at an amount greater
than the guarantee from P Ltd, P Ltd would not need to pay any amount to R Ltd.

28
The following information relating to the lease is available to both parties:

▪ 1 January 20.16 is the commencement date of the lease;


▪ R Ltd incurred initial direct costs of R5 000 related to the lease agreement and paid in cash;
▪ P Ltd incurred initial direct cost of R15 000 related to the lease and paid in cash;
▪ Annual lease payments, payable in arrears are R7 500 000; and
▪ The fair value of the building on commencement date is R51 200 000.

SOLUTION 13

The interest rate implicit in the lease (lessor’s perspective) is calculated as follows:

PV = - R51 205 000 (R51 200 000 fair value + R5 000 initial direct costs incurred by lessor)
N = 10
PMT = R7 500 000
FV = R28 000 000 (R25 000 000 residual value guarantee + R3 000 000 unguaranteed residual value
(R28 000 000 – R25 000 000)) I = ?;
I = 12,07%
P Ltd (lessee) will initially recognise the lease liability at the present value of the unpaid lease
payments using the above interest rate implicit in the lease of 12,07% (rounded), which is
R42 255 522 (FV = 0, N = 10, PMT = 7 500 000, I = 12,07%).

Whenever it is impracticable to determine this rate (remember that it is calculated from the
perspective of the lessor and all information may not necessarily be available to the lessee), the
lessee’s incremental borrowing rate of interest is used (refer to example 9.10 above). This is the
rate the lessee would have to pay to borrow over a similar term, and with a similar security, the
funds necessary to obtain an asset of a similar value to the right- of-use asset in a similar economic
environment

7.4.4 Subsequent measurement of leases: Lessee

STUDY

RIGHT-OF-USE OF ASSET LEASE LIABILITY


Subsequent measurement: Subsequent measurement:
After the commencement date of After commencement date, the lease liability is
the lease, the lessee will measure measured by:
the right-of-use asset applying the • Increasing the carrying amount to reflect interest on
cost model UNLESS another the lease liability;
measurement model applies. • Reducing the carrying amount to reflect the lease
payments (PMT) made; and

29
Cost • Remeasuring the carrying amount to reflect lease
• Right-of-use asset measured at modifications or revised in-substance fixed lease
cost. Less accumulated payments.
depreciation; The following will be included in profit / loss, UNLESS it
• Less accumulated impairment is included in the carrying amount of another asset:
(IAS 36); and • Interest on the lease liability; AND
• Adjusted for any remeasurement • Variable lease payments not included in the
of the lease liability (e.g. to reflect measurement of the liability in the period in which the
lease modifications or revised in- event or condition that triggers those payments occur.
substance fixed lease payments). DR Finance costs (P/L) xx
Other measurement models DR Lease liability (SFP) xx
Right-of-use asset measured at fair CR Bank (SFP) xx
value if: Payment of lease instalment on….
• Right-of-use assets meets IAS 40 OR
Investment property definition; DR Finance costs (P/L) xx
and CR Bank (SFP) xx
• Lessee accounts for investment DR Lease liability (SFP) xx
property on the fair value model.
CR Bank (SFP xx
• Revaluation Model: (Excluded Payment of lease instalment on….
from FAC2601)
DR Variable lease payments expenses (P/L) xx
CR Bank (SFP) xx
Payment of variable lease payments on …

A lease gives rise to a depreciation expense for the leased asset (e.g. depreciation on machinery) as well as a finance
cost for each accounting period. The depreciation policy for depreciable leased assets should be consistent with that
for depreciable assets which are owned, and the depreciation recognised shall be calculated in accordance with IAS
16 Property, plant and equipment, IAS 38 Intangible Assets and IAS 40 Investment Property.
The right-of-use asset is depreciated over its useful life if ownership transfers at the end of the lease term. If the
transfer of ownership is not certain, the right-of-use asset shall be depreciated over the shorter of its useful life or
lease term.

EXAMPLE 14: LESSEE: SUBSEQUENT MEASUREMENT

Assume the same facts as the first part of Example 11 and the following additional information:
▪ The lease does not include any options to extend or terminate the lease and ownership of the
office building does not transfer to P Ltd at the end of the lease term;
▪ P Ltd does not own another office building; and
▪ In terms of IAS 16, depreciation is calculated in accordance with the straight-line method over
the estimated useful life.

30
Amortisation table for first three years

Date PMT Interest, 12 % Capital Balance


R R R R
1 January 20.16 15 269 504
1 January 20.16: deposit 500 000 500 000 14 769 504
31 December 20.16 2 500 000 1 772 340 727 660 14 041 844
31 December 20.17 2 500 000 1 685 021 814 979 13 226 865

SOLUTION 14:

The journal entries for the year ended 31 December 20.16 and 20.17 will be as follows:
Dr Cr
R R
Journal entries:

31 December 20.16
Depreciation (P/L) (15 269 504 + 20 000 – 10 000)/10) 1 527 950
Accumulated depreciation: right-of-use asset: office building (SFP) 1 527 950
Depreciation for the year

Lease liability (SFP) 727 660


Interest expense (P/L) 1 772 340
Bank (SFP) 2 500 000
Payment of first instalment (AMORT 1)

31 December 20.17
Depreciation (P/L) 1 527 950
Accumulated depreciation: right-of-use asset: office building (SFP) 1 527 950
Depreciation for the year

Lease liability (SFP) 814 979


Interest expense (P/L) 1 685 021
Bank (SFP) 2 500 000
Payment of second instalment (AMORT 2)

Comments:

➢ The lease does not transfer ownership of the office building; therefore the useful life of the
right-of-use asset will be limited to the lease term of ten years.
➢ The depreciation charge can also be accounted for directly against the carrying amount of the
right-of-use asset. IFRS 16 only requires that the carrying amount at the beginning of the year
be reconciled to the carrying amount at the end of the year and does not require that the
carrying amount of the asset be split into the ‘cost’ and ‘accumulated depreciation’.

31
➢ If the payment dates of the lease contract and the reporting date of the entity do not coincide,
an interest expense accrual must be accounted for.
➢ When instalments are payable in advance, care should be taken regarding the finance charges,
as the finance charges are paid in a different period than what they are accrued in. This is due
to the first instalment, which is payable immediately, and only consists of capital. This will
result, for example, in the finance charges relating to the first period only being paid with the
second instalment.
The disclosure of the lease is illustrated in Example 9.15 below.

7.4.5 Reassessment of lease liability: lessee

STUDY

If lease payments change after the commencement date, the lease liability should be remeasured
to reflect such changes. The amount of the remeasurement of the lease liability is an adjustment to
the right-of-use asset, limited to Rnil (the remaining amount shall be recognised in profit or loss).

A lessee shall remeasure the lease liability by discounting such revised lease payments using an
unchanged discount rate, if:
• There is a change in the amounts expected to be payable under a residual value guarantee; or
• There is a change in future lease payments to reflect market rates (e.g. based on a market
rent review) or a change in an index or rate used to determine the lease payments.

EXAMPLE 15: REASSESSMENT OF LEASE LIABILITY

Assume the same facts as Example 14 except for the following additional information:
▪ The carrying amount of the lease liability on 31 December 20.17 is R13 226 865 (R14 769 504 –
R727 660 – R814 979) and the right-of-use asset is R12 223 604 (R15 279 504 – R1 527 950 –
R1 527 950).
▪ On 1 January 20.18 (after two years since the commencement of the lease), P Ltd estimated
that the amount payable under the residual value guarantee decreased to R1 000 000 (and
not R2 000 000 any longer).

Remeasurement of the lease liability:

PMT = R2 500 000


N = 8 (remaining term of lease)
I = 12% (unchanged discount rate)
FV = R1 000 000
PV = ? (R12 822 983)

32
The lease liability of R13 226 865 should now be adjusted to R12 822 983 to reflect the reassessment
(change in accounting estimate) and the liability is reduced by the difference of R403 882. Following
this adjustment, the interest expense for 20.18 and thereafter will be based on the revised liability.

The contra-account for the reassessment of the lease liability is the right-of-use asset. The revised
carrying amount of the right-of-use asset with now be R11 819 722 (R12 223 604 – R403 882).
Following this adjustment, the depreciation expense for 20.18 and thereafter will be based on the
revised carrying amount of the right-of-use asset.

P Ltd will process the following journals for the year ended 31 December 20.18:
Dr Cr
R R
Journal entries:

1 January 20.18
Lease liability (SFP) 403 882
Right-of-use asset (SFP) 403 882
Remeasurement of lease liability (R12 822 983 - R13 226 865)
31 December 20.18

Lease liability (SFP) 961 242


Interest expense (P/L) 1 538 758
Bank (SFP) 2 500 000
Payment of instalment (AMORT 1 – using new calc)

Depreciation (P/L) (SFP) 1 477 465


Accumulated depreciation: right-of-use asset: office 1 477 465
building (SFP)
Depreciation for the year
((R12 223 604 – R403 882)/8 years)

DISCLOSURE OF LEASES – LESSEE (EXAMPLE 14)

Notes for the year ended 31 December 20.16


1. Leases
1.1 Right--of-use assets:

Office building Total


R R
Carrying amount at the start of the period 0 xx xxx
Additions (including initial costs capitalised) 15 279 504 xx xxx
Depreciation (1 527 950) (xx xxx)
Adjustments for lease reassessments xx xxx xx xxx
Adjustments for lease modifications xx xxx xx xxx
Total xx xxx xx xxx

33
1.2 Lease liability:
20.16
Rand
Opening balance –
New leases entered into 15 269 504
Deposits paid (500 000)
Repayment of capital (727 660)
Adjustments for lease reassessments xx xxx
Adjustments for lease modifications xx xxx
Closing balance 14 041 844

Maturity analysis of lease payments to be paid at the reporting date:


20.16
Rand
Future lease payments (undiscounted) (based on P Ltd’s judgement of appropriate time bands)
– For 20.17 2 500 000
– For 20.18 2 500 000
– For 20.19 2 500 000
– For 20.20 2 500 000
– For 20.21 2 500 000
– Remaining years ((2 500 000 x 4 years) + 2 000 000) 12 000 000
Total future lease payments 24 500 000
Total future finance costs* (amort 2 – 10) (10 458 156)
Lease liability* 14 041 844
Long-term portion presented under non-current liabilities 13 226 865
Short-term portion presented under current liabilities (amort 2) 814 979
* Please note: Strictly speaking, this information is not necessarily required by IFRS 16, but is shown
here so that the subtotals tie up with amounts presented on the face of the statement of financial
position.

1.3 Potential future lease payments relating to periods following the exercise date of
termination options are summarised below:

Lease liabilities Payable during Payable


recognised 20.20 – 20.21 during 20.22 Total
(Discounted) (Undiscounted)
R R R R
Business segment xx xxx xx xxx xx xxx xx xxx
Brand A xxx xxx xxx xxx
Brand B xx xxx xx xxx xx xxx xx xxx
Total xx xxx xx xxx xx xxx xx xxx

34
1.4 Income and expenses related to leases
R
Income
Income from subleasing right-of-use assets xxx xxx
Gain from sale and leaseback xxx xxx
Expenses
Variable lease payments xx xxx
Short-term lease expense – recognition exemption xx xxx
Low value lease expense – recognition exemption xx xxx

P Ltd elected the recognition exemption on short-term leases of office equipment and low value
leases of office furniture

1.5 Total cash outflows relating to leases


R
Presented under financing activities
Cash payments for principal portion of lease liabilities 727 660
Presented under operating activities
Cash payments for interest portion of lease liabilities 1 772 340
Cash payments for short-term leases xx xxx
Cash payments for low value leases xx xxx
Cash payments for variable lease payments xx xxx
Total cash outflow relating to leases xxx xxx

2. Finance cost
R
Finance cost on financial liabilities xx xxx
Finance cost on lease liabilities 1 772 340
Other finance cost xx xxx
Borrowing cost capitalised (xx xxx)
Finance cost recognised in profit or loss xxx xxx

Borrowing cost has been capitalised to qualifying assets using a capitalisation rate of x,xx% p.a. The
portfolio of short-term leases to which P Ltd is committed at the end of 31 December 20.16 is
similar to the portfolio of short-term leases expenses recognised during the year.

What IFRS 16 refers to as “additional qualitative and quantitative information” about an entity’s
leasing activities necessary to meet the disclosure objective of IFRS 16 should also be included in
this note. This would include, but is not limited to, the following:
▪ the nature of the lessee’s leasing activities;
▪ future cash outflows to which the lessee is potentially exposed that are not reflected in the
measurement of lease liabilities, for example, exposure arising from variable lease payments
(IFRS 16.B49), extension and termination options (IFRS 16.B50), residual value guarantees
(IFRS 16.B51) and leases not yet commenced but to which the lessee is already committed; and
▪ restrictions imposed by leases.

35
The IFRS 16 references provided in brackets above provide additional information that the lessee
needs to disclose to satisfy the disclosure objective of this Standard.

7.5 PRESENTATION AND DISCLOSURE: LESSEE

IFRS 16 Leases requires an entity to disclose information relating to its leases for which it is a lessee
in a separate note or section in the financial statements. However, lessees are not required to
duplicate information disclosed elsewhere, provided that the information is cross-referenced in the
single note or separate section.

• A lessee must disclose the following amounts in a tabular format for the reporting period, unless
another format is more appropriate.
(a) Depreciation charge for right-of-use assets by class of underlying asset;
(b) Interest expense on lease liabilities;
(c) Expenses relating to variable payments not included in the measurement of the lease liabilities;
(d) Income from subleasing right-of-use assets;
(e) Total cash outflow for leases;
(f) Additions to right-of-use assets;
(g) Gains or losses arising from sale and leaseback transactions (only examinable on postgraduate
level); and
(h) The carrying amount of right-of-use assets at the end of the reporting period.

Please see 7.4.1 for the disclosure required when the lessee has elected to make use of recognition
exemptions for the lease of low-value assets and short-term leases.
• For right-of-use assets that meet the definition of investment property, a lessee will apply IAS
40 Investment property disclosure requirements. In this case, the disclosures above (a, d, f, h)
are not required.
• For right-of-use assets revalued amounts applying IAS 16 Property, plant and equipment, the
lessee will apply IAS 16 disclosure requirements relating to revalued assets. (Excluded from
FAC 2601). Revaluations are only dealt with regards to non-depreciable assets.
• A lessee must disclose a maturity analysis of lease liabilities by applying IFRS 7 Financial
instruments: Disclosures, separately from the maturity analyses of other financial liabilities that
show the remaining contractual maturities.
• A lessee shall disclose additional qualitative and quantitative information about its leasing
activities. This information may include, but is not limited to, information that help users of
financial statements to assess:
– The nature of the lessee’s leasing activities;
– Future cash outflows to which the lessee is potentially exposed that are not reflected in the
measurement of lease liabilities. This includes exposure arising from:
◊ Variable lease payments;
◊ Extension and termination options;
◊ Residual value guarantees; and
◊ Lease not yet commenced to which the lessee is committed.
– Restrictions or covenants imposed by leases; and
– Sale and leaseback transactions.

36
DISCLOSURE EXAMPLE – LESSEE
NOTES FOR THE YEAR ENDED 31 DECEMBER 20.16

1. Leases
Equip-
1.1 Right-of-use assets ment Total
R R
Carrying amount at beginning of year xxx Xxx
Additions xxx Xxx
Depreciation for the year xxx Xxx
Adjustments for lease remeasurements xxx Xxx
Adjustments for lease modifications xxx Xxx
Carrying amount at end of year xxx Xxx

1.2 Maturity analysis of future lease payments outstanding at the reporting date
20.16
Future lease payments (undiscounted) R
– For 20.17 Xxx
– For 20.18 Xxx
– For 20.19 Xxx
– For 20.20 Xxx
– Remaining years after 20.20 Xxx
Total future lease payments Xxx
Total future finance costs (xxx)
Lease liability Xxx
Short-term portion presented under current liabilities Xxx
Long-term portion presented under non-current liabilities Xxx

1.3 Potential future lease payments relating to periods following the exercise date of
termination options are summarised below:

Lease liabilities Payable during Payable


recognised 20.20 – 20.21 during 20.22 Total
(Discounted) (Undiscounted)
Business segment xxxx xxxx xxxx xxxx
Business segment A xxx xxx xxx xxx
Business segment B xxx xxx xxx xxx
Total xxxx xxxx xxxx xxxx

37
1.4 Income and expenses related to leases
R
Income
Income from subleasing right-of-use assets xxx
Gain from sale and leaseback (will only be tested on postgraduate level) xxx
Expenses
Variable lease payments xxx
Short-term lease expense – recognition exemption xxx
Low-value lease expense – recognition exemption xxx

X Ltd elected the recognition exemption on short-term leases of office equipment (example) and
low-value leases of office furniture (example).

2. Finance cost R
Finance cost on financial liabilities xxx
Finance cost on lease liabilities xxx
Other finance costs xxx
Borrowing cost capitalised (xxx)
Finance costs capitalised in profit or loss xxx

Borrowing cost has been capitalised to qualifying assets using a capitalisation rate of x,xx% per
annum. The portfolio of short-term leases to which X Ltd is committed at 31 December 20.16 is
similar to the portfolio of short-term lease expenses recognised during the year.

Disclosure of finance costs


Finance costs can be disclosed as a separate note as shown above or as part of the
profit before tax note.

7.6 COMPREHENSIVE EXAMPLES: LESSEE

EXAMPLE 16 – Low-value assets – recognition exemption

Acca Ltd entered into a contract with Beta SA Ltd to make use of a specialised printer and computer,
starting on 1 January 20.17. The contract meets the requirements of a lease in terms of IFRS 16
Leases. The asset is classified as a low-value asset. Acca Ltd makes use of the recognition exemption
in terms of IFRS 16 Leases, for all short-term and low-value assets.

The terms of the lease are as follows:


Lease term 4 years
Initial payment R1 440
Instalment payable monthly in arrears:
Months 1–24 R800
Months 25–48 R600

The lessee incurred R500 in legal fees for negotiating the lease for useful life of computer and the
printer: 3 years
38
REQUIRED
Discuss the accounting treatment of Acca Ltd for the year ended
31 December 20.17.
Calculate:

Equalised monthly instalments

1. Prepaid expense or accrued expense as at 31 December 20.17


2. Prepaid expense or accrued expense as at 31 December 20.18
(c) Disclose the low-value lease asset in the following notes to the annual financial statements of
Acca Ltd for the year ended 31 December 20.18.
• Prepaid expense
• IFRS 16 Leases (low-value assets)

Your answers must comply with the International Financial Reporting Standards (IFRS).
Ignore any income tax and VAT implications.
Round all amounts to the nearest rand.

SOLUTION 16

(a) Discussion of accounting treatment

As per IFRS 16 Leases, a lessee may elect to apply the recognition exemption to short- term leases
(less than 12 months) and low-value assets (less than $5 000). It is the policy of Acca Ltd to apply
the recognition exemption to all short-term and low-value assets.

As per IFRS 16 Leases, low-value assets are accounted for on a lease-by-lease basis.

Recognition

When the recognition exemption is applied, the lessee will recognise the lease payments on a low-
value asset lease as an expense on:
• A straight-line basis over the lease term; or
• Another systematic basis if it is more representative of the pattern of the lessee’s benefit.

The lessee will only consider the lease to be a new lease when:
• There is a lease modification; or
• There is any change in lease term.

Conclusion:

The lease payments for low-value assets must be recognised as an expense on a straight- line basis
over the lease term of four years, as Acca Ltd elected to make use of the recognition exemption.

39
(b) Calculations

1. The equalised monthly instalment is calculated as follows:


R
Initial payment 1 440
Instalments: 1 – 24 months (24 x 800) 19 200
Instalments: 25 – 48 months (24 x 600) 14 400
35 040
Equalisation of lease instalments per month (35 040/48) 730

The implication of the equalisation of lease payments is that a portion of lease payments will
be a prepaid expense or accrued expense in the statement of financial position.

2. At the end of the first year (31 December 20.17):


R
Initial payment 1 440
Instalments paid (12 x 800) 9 600
Total lease instalments paid in cash for the first year 11 040
Equalised lease expense per year (730 [C1] x 12 months) 760
Prepaid expense (asset) (SFP) (11 040 – 8 760) 2 280

The initial direct cost of R500 is expensed immediately in profit or loss (P/L)

3. At the end of the second year (31 December 20.18):


R
Instalments paid (12 x 800) 9 600
Equalised lease expense per year (730 [C1] x 12 months) 8 760
Prepaid expense (asset) (9 600 – 8 760) 840
Prepaid expense balance 20.17 2 280
Prepaid expense (asset) (SFP) 20.18 3 120

Prepaid expense versus accrued expense


A company will recognise a prepaid expense for lease payments when the actual
annual lease payment paid exceeds the annual equalised lease payment.

A company will recognise an accrued expense for lease payments when the
actual annual lease payment paid is less than the annual equalised lease
payment.

40
If the monthly instalments from example 3 were changed as follows:
Initial payment 1 440
1 – 24 months 600
25 – 48 months 800

The equalised lease payment would still be R730 per month; however, the equalised lease payment
of R8 760 (R730 x 12) would be more than the actual lease expense of R8 640 (1 440 + 600 x 12).
This difference of R120 is an accrued expense for the year ended 31 December 20.17.

(c) Disclosure

ACCA LTD
NOTES TO THE ANNUAL FINANCIAL STATEMENTS FOR THE YEAR ENDED 31 DECEMBER 20.18.

1. Prepaid expenses
20.18 20.17
R R
Low-value asset lease: prepaid expense [C2 & C3] 3 120 2 280

2. Leases

2.1 Income and expenses related to leases

Expenses
Low-value lease expense – recognition exemption [C2] 8 760 8 760

Acca Ltd elected the recognition exemption on low-value leases of office equipment.

Important
The computer and printer are not recognised as right-of-use assets. As a result, no
depreciation is recognised for the low-value assets.

EXAMPLE 17 – Contract is a lease in terms of IFRS 16

Scottle Ltd entered into a contract with Throttle Ltd on 1 January 20.17, whereby Throttle Ltd will
lease a machine for a period of four years. The cash selling price and fair value of the machine was
R400 000 on 1 January 20.17 and the instalments amount to R132 000 and are payable annually in
arrears. The interest rate implicit in the lease is 12.1104% per year. Ownership of the machine will
transfer to Throttle Ltd at the end of the lease term at no additional cost. The contract is classified
as a lease in terms of IFRS 16 Leases.

The machine has a useful life of five years and it is the company’s policy to depreciate the machine
over its useful life on the straight-line basis. The company financial year-end is 31 December.

41
REQUIRED
(a) Calculate the value of the right-of-use asset and the corresponding liability for
Throttle Ltd.
(b) Prepare the journal entries in the accounting records of Throttle Ltd for the
year ended 31 December 2017.

Your answers must comply with the International Financial Reporting Standards (IFRS). Round all
amounts to the nearest rand.

SOLUTION 17

(a) Calculations:
R
Right-of-use asset (machine)
Initial measurement of the lease liability *400 000
Initial direct costs of lessee –
Lease payments made at or before commencement date less any incentive received –
Estimate of dismantling costs –
400 000
Lease liability

Present value of lease payments not paid at lease commencement date *400 000
* The present value of the lease payments is calculated as follows:
N = 4 (1 payment per year x 4 years)
PMT = 132 000
FV = 0 + 0 (guaranteed and unguaranteed residual values)
I = 12,1104% p.a.
PV = ? 400 000

Exam hint
Please show calculations for all the amounts used in journals or disclosure so
that the marker can follow your calculations and award the marks!

(b) Journals as at:


DR CR
1 January 2017 R R
Right-of-use asset: Machine (SFP) 400 000
Lease liability (SFP) 400 000
Recognition of right-of-use asset and lease liability

31 December 2017
Depreciation (P/L) 80 000
Accumulated depreciation (SFP) (400 000/5) 80 000
Recognition of depreciation for the year

42
Finance costs (P/L) 48 442
Lease liability (SFP) (400 000 x 12,1104% x 12/12) 48 442
Recognition of finance costs that accrued from
1 January 2017 – 31 December 2017

Lease liability (SFP) 132 000


Bank (SFP) 132 000
Payment of lease instalment on 31 December 20.17

OR

Finance costs (P/L) 48 442


Lease Liability (SFP) 83 558
Bank (SFP) 132 000
Payment of lease instalment on 31 December 20.17

EXAMPLE 18 – amortisation table

Bun Ltd entered into a contract with Doughnut SA Ltd on 1 March 20.16 whereby Doughnut SA Ltd
will lease a specialised oven to Bun Ltd. The contract meets the requirements of a lease in terms of
IFRS 16 Leases.

The terms of the lease are as follows:


Lease term 4 years
Initial payment R144 000
Instalment payable monthly in arrears R20 000
Useful life of oven 6 years
The lessee’s incremental borrowing rate on 1 March 20.16: 8% per annum

The lessee incurred R4 800 in legal fees for negotiating the lease. It is impracticable for the lessee
to calculate the costs incurred by the lessor and as a result the interest rate implicit in the lease
cannot be calculated.

REQUIRED

(a) Calculate the value of the right-of-use asset and the corresponding lease
liability for Bun Ltd at initial recognition.
(b) Prepare the journal entries in the accounting records of Bun Ltd for the year
ended 28 February 20.17.

Your answers must comply with the International Financial Reporting Standards (IFRS). Round all
amounts to the nearest rand.

43
SOLUTION 18

(a) Calculations:

Right-of-use asset (Oven) R


Initial measurement of the lease liability *819 238
Initial direct costs of the lessee 4 800
Lease payments made at or before commencement date less any incentive
received 144 000
Estimate of dismantling costs –
968 038
Lease liability

Present value of lease payments not paid at lease commencement date *819 238

* The present value of the lease payments is calculated as follows:

N = 48 (12 payments per year x 4 years)


PMT = 20 000
FV = 0 (residual values guarantee)
I = 8% p.a.
PV = ? 819 238

(b) Journals for the year ended 28 February 20.17:


DR CR
1 March 20.16 R R
Right-of-use asset: Oven (SFP) 819 238
Lease liability (SFP) 819 238
Recognise asset and liability

Right-of-use asset: Oven (SFP) 4 800


Bank (SFP) 4 800
Recognise initial direct costs (legal fees) as part of cost of asset

Right-of-use asset: Oven (SFP) 144 000


Bank (SFP) 144 000
Recognise deposit as part of cost of asset

OR

Right-of-use asset: Oven (SFP) 968 038


Lease liability (SFP) 819 238
Bank (SFP) (4 800 + 144 400) 148 800
Recognise asset, liability and cost of asset paid in cash

44
28 February 20.17
Depreciation (P/L) 242 010
Accumulated depreciation (SFP) (968 038/4) 242 010
Depreciation for the year

Finance costs (P/L) [C1] 58 998


Lease liability (SFP) [C1] 58 998
Total finance costs for the year

Lease liability (SFP) [C1] 240 000


Bank (SFP) [C1] 240 000
The total of 12 lease payments for the year

OR

Finance costs (P/L) [C1] 58 998


Lease liability (SFP) [C1] 181 002
Bank (SFP) [C1] 240 000
The total of 12 lease payments for the year

C1. Amortisation table


Payment date Instalment Capital Interest at 8% Balance
R R R R
1 March 20.16 819 238
31 March 20.16 20 000 14 538 5 462 804 700
30 April 20.16 20 000 14 635 5 365 790 064
31 May 20.16 20 000 14 733 5 267 775 331
30 June 20.16 20 000 14 831 5 169 760 500
31 July 20.16 20 000 14 930 5 070 745 570
31 August 20.16 20 000 15 030 4 970 730 541
30 September 20.16 20 000 15 130 4 870 715 411
31 October 20.16 20 000 15 231 4 769 700 180
30 November 20.16 20 000 15 332 4 668 684 848
31 December 20.16 20 000 15 434 4 566 669 414
31 January 20.17 20 000 15 537 4 463 653 877
28 February 20.17 20 000 15 641 4 359 638 236
240 000 181 002 58 998

45
Amortisation table

The complete amortisation table includes 48 months/payments. Only the first


12 months were done as only the first 12 months are required to do the journals
at 28 February 20.17.

The totals of the amortisation table above can also be retrieved from your financial calculator as
follows:

Interest 58 988, Capital 181 002, Balance 638 236

SHARP EL738 HP SHARP (OLD)


Amort – P1 = 1 enter 1 Amort 1 P1/P2 12 account = capital
Arrow down 12 enter 1 P1/P2 12 account = interest
P2 = 12 enter 12 amort = balance

EXAMPLE 19 – Comprehensive

A manufacturing company, Rubble Ltd, entered into a contract on 1 January 20.16 whereby two
machines with a total cash selling price of R412 500 would be leased from a finance company. The
contract contains a lease in terms of IFRS 16 Leases. The lessor did not incur any initial direct costs.

The period of the lease is three years and the lease payments of R49 205 are payable quarterly in
arrears. Rubble Ltd will obtain ownership of the machines at the end of the lease term at no
additional cost. Rubble Ltd paid R20 000 in legal fees for negotiating the lease. The interest rate
implicit in the lease is 24,00478% per annum.

The profit before tax of Rubble Ltd for the year ended 31 December 20.16 before the above lease
transactions, amounted to R300 000. The two machines have Rnil residual value and will be
depreciated over their expected useful lives of four years using the straight-line method.
The company’s reporting period ends on 31 December each year.

REQUIRED
Prepare the journal entries of Rubble Ltd for the abovementioned lease for
(a) the financial year ended 31 December 20.16.
(b) Prepare the notes to the financial statements of Rubble Ltd at
31 December 20.16 (excluding the accounting policy notes and IFRS 7
Disclosure) in respect of the above lease. Comparative figures are not
required.

Your answers must comply with the International Financial Reporting Standards (IFRS). Round all
amounts to the nearest rand.

46
SOLUTION 19

(a) Journal entries for the year ended 31 December 20.16


Dr Cr
1 January 20.16 R R
Right-of-use assets: Machinery (SFP) [C2] 20 000
Bank (SFP) 20 000
Capitalise legal fees to the cost of the machinery

Right-of-use asset: Machinery (SFP) [C2] 412 500


Lease liability (SFP) 412 500
Recognise right-of-use assets and the lease liability

Lease liability (SFP) [C4] 106 960


Finance costs (P/L) [C4] 89 860
Bank (SFP) (4 x 49 205) [C4] 196 820
Recognition of four instalments paid during 20.16

Depreciation (P/L) ((412 500 + 20 000)/4 years) 108 125


Accumulated depreciation (SFP) 108 125
Recognise depreciation on right-of-use assets

(b) Disclosure

RUBBLE LTD
NOTES FOR THE YEAR ENDED 31 DECEMBER 20.16

1. Profit before tax


20.16
Profit before tax is stated after taking into account the following items: R
Expenses
Depreciation: Right-of-use assets [C5] 108 125

2. Finance costs

Finance cost on lease liabilities [C4] 89 860

47
3. Leases
20.16 20.16
3.1 Right-of-use assets Machinery Total
R R
Carrying amount at beginning of year – –
Additions [C2] 432 500 432 500
Depreciation for the year [C5] (108 125) (108 125)
Adjustments for lease re-measurements – –
Adjustments for lease modifications – –
Carrying amount at end of year 324 375 324 375

3.2 Maturity analysis of future lease payments outstanding at the reporting date
R
Future lease payments (undiscounted) 20.16
– For the year ended 31 December 20.17 (49 205 x 4) [C4] 196 820
– For the year ended 31 December 20.18 (49 205 x 4) [C4] 196 820
– For the year ended 31 December 20.19 –
– For the year ended 31 December 20.20 –
– Remaining years after 31 December 20.20 –
Total future lease payments 393 640
Total future finance costs [C4] (88 100)
Lease liability [C4] 305 540
Short-term portion presented under current liabilities [C4] 135 042
Long-term portion presented under non-current liabilities [C4] 170 498

Calculations

C1. Interest rate implicit in the lease

The interest rate implicit in the lease is given as 24,00478% per annum.

C2. Right-of-use asset and lease liability

Right-of-use assets (Machinery) R


Initial amount of the lease liability [C3] 412 500
Initial direct costs of the lessee – legal fees 20 000
Lease payments made at or before commencement date –
432 500
Lease liability –
Present value of lease payments at lease commencement date [C3] 412 500

48
C3. Present value of lease payments

N = 12 (4 payment per year x 3 years)


PMT = 49 205
FV = 0 (residual value guarantee)
I = 24,00478%
PV = ? 412 500

C4. Amortisation table


Interest at Closing
Payment dates Instalments Capital 24,00478% balance
R R R R
01.01.20.16 412 500
31.03.20.16 49 205 24 450 Finance
24 755 388 050
30.06.20.16 49 205 25 917 cost in 23 288 362 133
P/L
30.09.20.16 49 205 27 472 = 89 860 21 733 334 661
31.12.20.16 49 205 29 121 20 084 305 540
(year-end)
31.03.20.17 Future 49 205 30 869 18 336 274 671
lease
30.06.20.17 payments 49 205 32 721 16 484 241 950
20.17
30.09.20.17 =196 820 49 205 34 685 14 520 207 265
Future
31.12.20.17 49 205 36 767 finance 12 438 170 498
costs
31.03.20.18 Future 49 205 38 973 = 88 100 10 202 131 525
lease
30.06.20.18 payments
49 205 41 312 7 893 90 213
30.09.20.18 20.18 49 205 43 791 5 414 46 422
=196 820
31.12.20.18 49 205 46 422 2 783 -
590 460 412 500 177 960

Use of calculator for amortisation table


Remember the amortisation table is just a calculation. Therefore, remember to
use it in your disclosure. If it is not used in your answer, no marks will be awarded.

All the amounts in the amortisation table can be retrieved from you calculator.

49
Please refer to your calculator manual and ensure that you can retrieve the interest, capital and
capital balance from your calculator. For example, the finance cost for the year (R89 860) can be
retrieved from your calculator by calculating it as follows:

HP: 1 (Input) 4 (2nd function) (Amort) = interest, = capital, = capital balance OR


SHARP EL 738: Amort payments 1–4.

If you only make use of your calculator during the exam, write down the inputs and the steps you
used to obtain the amounts disclosed.

C5. Accounting profit


R
Accounting profit (given) 300 000
Depreciation (412 500 + 20 000)/4 years) (108 125)
Finance costs [C4] (89 860)
Adjusted accounting profit 102 015

Legal fees of R20 000


The legal fees were capitalised (added) to the asset. Therefore, depreciation of
R108 125 (see note 3 above) includes R5 000 that relates to legal fees (20 000 /
4 years). The balance of the legal fees that will form part of future depreciation
amounts to R15 000 (R20 000 – R5 000).

SCHEDULE A: Basic examples on how to prepare an amortisation table

An amortisation table is prepared to calculate the split between capital and interest of the lease
instalments. Please make sure that you can calculate the interest rate implicit in the lease on your
financial calculator. You must also be able to calculate the amortisation table manually and to
extract the numbers of the amortisation table from your financial calculator.

The four examples below will be used to illustrate important principles when preparing an
amortisation table.

EXAMPLE 20

Payments in arrears

Acca Ltd entered into a finance lease agreement with Beta Ltd on 1 January 2017, whereby Beta Ltd
will lease a machine from Acca Ltd for a period of two years. The cash selling price and fair value of
the machine amounted to R40 000 on 1 January 2017. The instalments amount to R13 200 each and
are payable half-yearly in arrears. Acca Ltd didn’t incur any initial direct costs. Ownership of the
machine will transfer to Beta Ltd at the end of the lease term at no additional cost. The financial
year-end of Acca Ltd is 31 December.

50
SOLUTION 20
REQUIRED

Prepare the amortisation table that Acca Ltd will use to account for the lease of
the machine.

In this example 1, Acca Ltd is the lessor and Beta Ltd is the lessee.

Step 1: Calculate the interest rate implicit in the lease.

Set HP and Sharp EL-738 calculators on 2P/YR (as there are two instalments payable per year)

N = 4 (2 payments per year x 2 years)


PV = (40 000 + 0) (fair value + initial direct costs of the lessor)
PMT = 13 200*
FV = 0* + 0* (guaranteed and unguaranteed residual values)
I = ? 24,22% per annum

(Sharp EL-738 will calculate 12,11% per six months 12,11% x 2 = 24,22% for 12 months)

* Important
The amounts for “PMT” and “FV” entered into your financial calculator represent
cash in- flows for Acca Ltd and must have the same signs, for example, they must
both be positive or negative. The “PV” represents the outflow of the finance
provided for the machine to Acca Ltd from Beta Ltd and must have the opposite
sign of the “PMT” and “FV”.

Step 2: Determine the initial recognition amount.

Recognise the asset and liability at the lower of the:

– present value of the minimum lease payments discounted at the interest rate implicit in the
lease (R40 000), or
N = 4 (2 payments per year x 2 years)
I = 24,22% per annum (calculated in step 1)
PMT = 13 200
FV = 0 (guaranteed residual value only)
PV = ? (40 000)
– fair value of the leased property (R40 000).

51
Comment
The present value of the minimum lease payments is equal to the fair value of
R40 000. As a result, the machine and the finance lease liability will both be
recognised at R40 000. Consequently, the amortisation table is prepared at
R40 000.

Step 3: Prepare the amortisation table for R40 000.

Interest at Outstanding
Payment date Instalment 24,22% p.a. Capital balance
R R R R
1 January 20.17 40 000
30 June 20.17 13 200 a 4 844 b 8 356 c 31 644
31 December 20.17 13 200 3 832 9 368 22 276
30 June 20.18 13 200 2 698 10 502 11 774
31 December 20.18 13 200 1 426 11 774 –
52 800 12 800 40 000

a 40 000 x 24,22% x 6/12 = 4 844 or 40 000 x 12,11% = 4 844


b 13 200 – 4 844 = 8 356
c 40 000 – 8 356 = 31 644

EXAMPLE 21

Payments in advance
The same information applies as in example 7, except that the instalments are payable half-yearly
in advance.

REQUIRED

Prepare the amortisation table that Acca Ltd will use to account for the lease of
the machine.

52
SOLUTION 21

Step 1: Calculate the interest rate implicit in the lease.

IMPORTANT: Set your calculator on BEGIN mode, as instalments are payable at the beginning of
every six months.

Set HP and Sharp EL-738 calculators on 2P/YR (as there are two instalments payable per year)

N = 4 (2 payments per year x 2 years)


PV = (40 000 + 0) (fair value + initial costs of the lessor)
PMT = 13 200
FV = 0 + 0 (guaranteed and unguaranteed residual values)
I = ? 44,77% per annum

(Sharp EL-738 will calculate 22,38% per six months. 22,38% x 2 = 44,77% for 12 months)

Step 2: Determine the initial recognition amount.

Recognise the asset and liability at the lower of the:

– present value of the minimum lease payments discounted at the interest rate implicit in the
lease (R40 000), or
N = 4 (2 payments per year x 2 years)
I = 44,77% per annum (calculated in step 1)
PMT = 13 200
FV = 0 (guaranteed residual value only)
PV = ? (40 000)
– fair value of the leased property (R40 000).

Step 3: Prepare the amortisation table for R40 000.


Interest at Outstanding
Payment date Instalment 44,77% p.a. Capital balance
R R R R
1 January 20.17 40 000
1 January 20.17 13 200 – 13 200 a 26 800
1 July 20.17 13 200 b 5 999 c 7 201 19 599
1 January 20.18 13 200 4 387 8 813 10 786
1 July 20.18 13 200 2 414 10 786 –
52 800 12 800 40 000
a 40 000 – 13 200 = 26 800
b 26 800 x 44,77% x 6/12 = 5 999 or 26 800 x 22,38% = 5 999
c 13 200 – 5 999 = 7 201

53
Comments on Example 7 and Example 8
Compare the amortisation tables of example 7 and example 8. Note the
differences between the interest rates and payment dates as a result of the
instalments being payable in arrears or in advance.

The payment dates are different for example 8, as the instalments are payable at the beginning of
the six-month period. The lease was entered into on 1 January 20.17 and in example 8, the first
payment was made on the same date. As a result, the first instalment paid on 1 January 20.14
comprises capital only, because no time has passed for interest to accrue.

The interest rates in both example 7 and example 8 were rounded off to two decimals. In a question
you will normally be instructed to round off the interest rate to a specific number of decimals, for
example, round off the interest rate to two decimals.

EXAMPLE 22

Payments in arrears

Example 9 contains the same information as example 7, except that the financial year-end of
Acca Ltd is 30 April and not 31 December.

REQUIRED

Prepare the amortisation table that Acca Ltd will use to account for the lease of
the machine.

SOLUTION 22

Step 1 and Step 2 are the same as for example 7.

Step 3: Prepare the amortisation table for R40 000.

Payments do not coincide with year-end


The payments are made on 30 June and 31 December and the financial year-end
is 30 April. Although no instalments are made at year-end, interest is accrued at
year-end.

54
Interest @ Outstanding
Payment date Instalment 24,22% p.a. Capital balance
R R R R
1 January 20.17 40 000
30 April 20.17 – a 3 229 – b 43 229
30 June 20.17 13 200 c 1 615 d 8 356 e 31 644
31 December 20.17 13 200 f 3 832 g 9 368 h 22 276
30 April 20.18 – i 1 799 – j 24 075
30 June 20.18 13 200 k 899 l 10 502 m 11 774
31 December 20.18 13 200 1 426 11 774
52 800 12 800 40 000

a 40 000 x 24,22% x 4/12 = 3 229 or 40 000 x 12,11% x 4/6 = 3 229


b 40 000 + 3 229 = 43 229. R43 229 is the capital outstanding and the interest accrued as at year-
end combined. This amount is only calculated for disclosure in the financial statements and must
not be used to calculate interest in the amortisation table.
c 40 000 x 24,22% x 2/12 = 1 615 or 40 000 x 12,11% x 2/6 = 1 615
Please note that the interest from 1 January 20.17 until 30 June 20.17 totals R4 844 and is split
into R3 229 (for four months) and R1 615 (for two months).
d 13 200 – (3 229 + 1 615) = 8 356
e 40 000 – 8 356 = 31 644
f 31 644 x 24,22% x 6/12 = 3 832 or 31 644 x 12,11% x 6/6 = 3 832
g 13 200 – 3 832 = 9 368
h 31 644 – 9 368 = 22 276
i 22 276 x 24,22% x 4/12 = 1 799 or 22 276 x 12,11% x 4/6 = 1 799
j 22 276 + 1 799 = 24 075. R24 075 is the capital outstanding and the interest accrued as at year-
end. This amount is only calculated for disclosure in the financial statements and must not be
used to calculate interest in the amortisation table.
k 22 276 x 24,22% x 2/12 = 899 or 22 276 x 12,11% x 4/6 = 899
l 13 200 – (1 799 + 899) = 10 502
m 22 276 – 10 502 = 11 774

55
EXAMPLE 23

Payments in advance

Example 10 contains the same information as example 8, except that the financial year- end of
Acca Ltd is 30 April and not 31 December.

REQUIRED

Prepare the amortisation table that Acca Ltd will use to account for the lease of
the machine.

SOLUTION 23

Step 1 and Step 2 are the same as for example 8.

Step 3: Prepare the amortisation table for R40 000.

Comment

The payments are made on 30 June and 31 December and the year-end is 30 April.
No instalment is paid at year-end and only interest has accrued at year-end.

Interest at Outstanding
Payment date Instalment 44,77% p.a. Capital balance
R R R R
1 January 20.17 40 000
1 January 20.17 13 200 – a 13 200 b 26 800
30 April 20.17 – c 3 999 - d 30 799
1 July 20.17 13 200 e 2 000 f 7 201 g 19 599
1 January 20.18 13 200 h 4 387 i 8 813 j 10 786
30 April 20.18 – k 1 609 – l 12 395
1 July 20.18 13 200 m 805 n 10 786 –
52 800 12 800 40 000

a The full instalment of R13 200 is capital.


b 40 000 – 13 200 = 26 800
c 26 800 x 44,77% x 4/12 = 3 999 OR 26 800 x 22,38% x 4/6 = 3 999
d 26 800 + 3 999 = 30 799. R30 799 is the capital outstanding and the interest accrued as at year-
end. This amount is only calculated for disclosure in the financial statements and must not be
used to calculate interest in the amortisation table.

56
e 26 800 x 44,77% x 2/12 = 2 000 or 26 800 x 22,38% x 2/6 = 2 000
Please note that the interest from 1 January 20.17 until 30 June 20.17 totals R5 999 and is split
into R3 999 (for 4 months) and R2 000 (for 2 months).
f 13 200 – (3 999 + 2 000) = 7 201
g 26 800 – 7 201 = 19 599
h 19 599 x 44,77% x 6/12 = 4 387 or 19 599 x 22,38% x 6/6 = 4 387
i 13 200 – 4 387 = 8 813
j 19 599 – 8 813 = 10 786
k 10 786 x 44,77% x 4/12 = 1 609 or 10 786 x 22,38% x 4/6 = 1 609
l 10 786 + 1 609 = 12 395. R12 395 is the capital outstanding and the interest accrued as at year-
end. This amount is only calculated for disclosure in the financial statements and must not be
used to calculate interest in the amortisation table.
m 10 786 x 44,77% x 2/12 = 805 or 10 786 x 22,38% x 2/6 = 805
n 13 200 – (1 609 + 805) = 10 786
o 10 786 – 10 786 = 0

57

You might also like